Line l has a slope of −3. The line through which of the following pair of points is perpendicular to l?

Answers

Answer 1

Answer:

The slope of the perpendicular line will 1/3.

Step-by-step explanation:


Related Questions

Find two integers whose sum is 1 and product is -2
TWO INTERGES

Answers

Answer:

2 and -1

Step-by-step explanation:

sum = 2 + (-1)

= 1

product = 2 × ( -1)

= -2

the length of a pond is 1700 CM breadth is 14m and height is 1000 CM if a point is half filled calculate the volume of a water in the pond ​

Answers

Answer:

1190 m^3

Step-by-step explanation:

l = 1700 cm = 17 m

b = 14 m

h = 1000 cm = 10 m

Total volume = l × b × h

= 17 × 14× 10

= 2380 m^3

since it is half filled ,

Volume is half , so,

volume of water in pond = 2380 ÷ 2

= 1190 m^3

Factorise a² - b²
.......

Answers

(a-b)(a+b)

Regards,
ArmyCee:)

[tex]\longrightarrow{\green{ (a + b)(a - b) }}[/tex] 

[tex]\sf \bf {\boxed {\mathbb {STEP-BY-STEP\:EXPLANATION:}}}[/tex]

[tex] ={a}^{2} - {b}^{2} [/tex]

[tex] = (a + b)(a - b)[/tex]

[tex]\bold{ \green{ \star{ \orange{Mystique35}}}}⋆[/tex]

Al lanzar un dado dos veces consecutivas. ¿Qué probabilidad hay de obtener primero un 3 y luego un numero par?

Answers

Answer:

The probability is 1/12.

Step-by-step explanation:

Number of elements in sample space is 6 . Even numbers are 2, 4 and 6 so the there are 3 three even numbers.

So, the probability of getting 3 on the first chance and then an even number in the second chance is

[tex]P = \frac{1}{6}\times \frac{3}{6}\\\\P = \frac{1}{12}[/tex]

the average length of string a and string d is 10x cm. the average length of string b and string c is 8x cm . the average length of strings b , c , d is 6x cm . find the length of string a in terms of x

Answers

Answer:

[tex]a = 18x[/tex]

Step-by-step explanation:

Given

[tex]\frac{1}{2}(a + d) = 10x[/tex]

[tex]\frac{1}{2}(b + c) = 8x[/tex]

[tex]\frac{1}{3}(b + c+d) = 6x[/tex]

Required

The value of (a)

We have:

[tex]\frac{1}{2}(a + d) = 10x[/tex] --- multiply by 2

[tex]\frac{1}{2}(b + c) = 8x[/tex] --- multiply by 2

[tex]\frac{1}{3}(b + c+d) = 6x[/tex] --- multiply by 3

So, we have:

[tex]\frac{1}{2}(a + d) = 10x[/tex]

[tex]a + d = 20x[/tex]

[tex]\frac{1}{2}(b + c) = 8x[/tex]

[tex]b + c = 16x[/tex]

[tex]\frac{1}{3}(b + c+d) = 6x[/tex]

[tex]b + c + d= 18x[/tex]

Substitute [tex]b + c = 16x[/tex] in [tex]b + c + d= 18x[/tex]

[tex]16x + d = 18x[/tex]

Solve for d

[tex]d = 18x - 16x[/tex]

[tex]d = 2x[/tex]

Substitute [tex]d = 2x[/tex] in [tex]a + d = 20x[/tex]

[tex]a + 2x = 20x[/tex]

Solve for (a)

[tex]a = 20x - 2x[/tex]

[tex]a = 18x[/tex]

Jada asked some students at her school how many hours they spent watching television last week, to the nearest hour. Here are a box plot and a histogram for the data she collected.

If anyone trolls imma be very sad :(

Answers

Answer:

100

Step-by-step explanation:

Based on the histogram given :

The vertical axis gives the number of student in the school :

For the range :

0 - 5 = 40

5 - 10 = 30

10 - 15 = 20

15 - 20 = 5

20 - 25 = 3

25 - 30 = 2

Taking the sum :

(40 + 30 + 20 + 5 + 3 + 2) = 100

explain by step by step pls :( if u type something wrong ill report u

Answers

Answer:

∠ C = ∠BCD = 30°

Step-by-step explanation:

∠ EDF = ∠GFC  = 110°                [ corresponding angles ]

Now consider triangle BCF

∠FBC  + ∠ABC = 180°                [ straight line angle ]

∠FBC  + 100° = 180°

∠FBC  = 180 - 100 = 80°

∠BFC + ∠GFC = 180°                  [ straight line angle ]

∠BFC + 110° = 180°

∠BFC = 180 - 110 = 70°

Sum of angles of triangle is 180°

Therefore , in triangle BCF

That is ,

       ∠F + ∠B + ∠ C  = 180°

          70° + 80° + ∠C = 180°

           ∠C = 180 - 150 = 30°            

       

WILL GIVE BRAINLIEST
Hannah and Han are each trying to solve the equation x² – 8x + 26 = 0. They know that
x = -1 are i& - i, but they are not sure how to use this information to solve for x in their
equation.
Part 1- Here is Hannah's work:
x? - 8x + 26 = 0
X? – 8x = -26
Show Hannah how
to finish her work using completing the square and complex numbers.
Part 2- Han decides to solve the equation using the quadratic
formula. Here is the beginning of his
work
-b+V62-4ac
-(-8)+7-8)2–401|(26)
Finish using the quadratic formula. Simplify the final answer as much as possible.

Answers

Part one:

[tex]x^2-8x=-26[/tex]

Rewrite in the form [tex](x+a)^{2} =b[/tex]

[tex]\left(x-4\right)^2=-10[/tex]

[tex]\mathrm{For\:}f^2\left(x\right)=a\mathrm{\:the\:solutions\:are\:}f\left(x\right)=\sqrt{a},\:-\sqrt{a}[/tex]

Solve [tex]x-4=\sqrt{-10} : x=\sqrt{10} i+4[/tex]

Solve [tex]x-4=\sqrt{-10} : x=-\sqrt{10} i+4[/tex]

[tex]x=\sqrt{10}i+4,\:x=-\sqrt{10}i+4[/tex]

Part two:

[tex]x=\frac{-\left(-8\right)\pm \sqrt{\left(-8\right)^2-4\cdot \:1\cdot \:26}}{2\cdot \:1}[/tex]

Simplify [tex]\sqrt{\left(-8\right)^2-4\cdot \:1\cdot \:26}}: 2\sqrt{10} i[/tex]

[tex]=\frac{-\left(-8\right)\pm \:2\sqrt{10}i}{2\cdot \:1}[/tex]

Separate solutions

[tex]x_1=\frac{-\left(-8\right)+2\sqrt{10}i}{2\cdot \:1},\:x_2=\frac{-\left(-8\right)-2\sqrt{10}i}{2\cdot \:1}[/tex]

[tex]\frac{-(-8)+2\sqrt{10}i }{2*1} :4+\sqrt{10}i[/tex]

[tex]\frac{-(-8)+2\sqrt{10}i }{2*1} :4-\sqrt{10}i[/tex]

[tex]x=4+\sqrt{10}i,\:x=4-\sqrt{10}i[/tex]

The solutions are:-

[tex]x=4+\sqrt{10i}\\\\x=4-\sqrt{10i}[/tex]

What is the equation?

The definition of an equation is a mathematical statement that shows that two mathematical expressions are equal.

Here given equation is

[tex]x^2- 8x + 26 = 0\\\\x^2-8x=-26\\\\(x-4)^2=-10\\\\[/tex]

[tex](x-4)=[/tex]±[tex]\sqrt{-10}[/tex]

[tex]x=\sqrt{10}i+4\\\\x=-\sqrt{10}i+4[/tex]

So,

[tex]x=\frac{-b+-\sqrt{b^2-4ac}}{2a}\\\\x=\frac{8+-\sqrt{(-8)^2-4(1)(26)}}{2(1)}\\\\x=\frac{8+-\sqrt{(-8)^2-4(1)(26)}}{2(1)}\\\\x=\frac{8+-\sqrt{64-104}}{2}\\\\x=\frac{8+-2\sqrt{10i}}{2}\\\\x=\frac{2(4+-\sqrt{10i})}{2}\\\\x=4+\sqrt{10i}\\\\x=4-\sqrt{10i}[/tex]

Hence, the solutions are:-

[tex]x=4+\sqrt{10i}\\\\x=4-\sqrt{10i}[/tex]

To know more about the equation

https://brainly.com/question/12788590

#SPJ2

Can someone tell me if this is the correct choice?

Answers

Answer:

Step-by-step explanation:

It's not the correct choice. If a root is given, that is the same thing as the solution. We go backwards from a solution to a factor, which is what you need to do here.

If x = 2i is the solution, then the factor is

(x - 2i). Likewise, with the other one.

If x = 3i is the solution, then the factor is

(x - 3i). The leading coefficient of 1 just sits outside the first set of parenthesis, and because multiplication is commutative, it doesn't matter which set you put first. Thus, the one you want looks like this:

f(x) = (x - 2i)(x - 3i) or

f(x) = (x - 3i)(x - 2i).

Find the length of the missing

Answers

Answer:

12

Step-by-step explanation:

using pythagoras theorem

here 15 is hypotenuse since it is opposite 0f 90 degree

9 and x are the other smaller sides of a triangle

according to pythagoras thorem the sum of square of two smaller sides of a triangle is equal to the square of hypotenuse. So,

9^2 + x^2 = 15^2

81 + x^2 = 225

x^2 = 225 - 81

x^2 = 144

x = [tex]\sqrt{144[/tex]

x = 12

Can someone help me?It's urgent and thank you!

Answers

Answer:

y = square root x2 - 5

Step-by-step explanation:

y=[tex]\sqrt{x} -5[/tex]

(the top option)

Tell whether the equation Y=Y true or false

Answers

Answer:

its true trust biggie on this one fam

Step-by-step explanation:

its true broski

5. What is x in the diagram?

Answers

Answer:

B

Step-by-step explanation:

we rule out all other 3 by the simple fact that the side is 9

the answer is B hope this helps

Helppp and explain pleaseeeeee!!!!!!

Answers

7/3 is your answer!

Help please guys if you don’t mind

Answers

Answer:

first

2/5m + 8/5

second

22/5

third

11

Triangle HIJ is similar to triangle KLM. Find the measure of side MK. Round your answer to the nearest tenth.

Answers

Answer:

[tex]MK\approx 87.7[/tex]

Step-by-step explanation:

By definition, similar polygons have corresponding sides in a constant proportion. Therefore, we can set up the following proportion (ratio of corresponding sides) to solve for [tex]MK[/tex]:

[tex]\frac{20}{13}=\frac{MK}{57},\\MK=\frac{57\cdot 20}{13}\approx \boxed{87.7}[/tex]

Answer:

87.7

Step-by-step explanation:

Like stated previously similar triangle have side lengths with common ratios

*Create a proportionality to solve for MK*

57/13 = MK / 20

Now solve for MK

Multiply each side by 20

57/13 * 20 = 87.7 ( rounded )

MK/20 * 20 = MK

We're left with MK = 87.7

How do I do this?? TnT

Answers

Answer:

x = 4

Step-by-step explanation:

Corresponding angles are congruent

4x + 44 = 6x + 36

44 = 2x + 36

8 = 2x

x = 4

A scale model of a sculpture has a scale of 2:53
The height of the model is 20cm.
Find the height of the actual sculpture.
Give your answer in m.

Answers

Answer:

ok so we can simpley this is 1:106

so we multiply

20*106=2120 this is cm so we make it meters...

21.2meters!!!

Hope This Helps!!!

A steamer travels 36 km upstream and 32 km downstream in 6.5 hours. The same steamer travels 4 km upstream and 40 km downstream in 180 minutes. Determine the steamer's speed in still water and the stream's speed.

Answers

Answer:

The streamer's speed in still water is 90.23 km/h while the stream's speed is 33.33 km/h

Step-by-step explanation:

Let v = streamer's speed in still water and v' = stream's speed. His speed upstream is V = v + v' and his speed downstream is V' = v - v'.

Since he travels 36 km upstream, the time taken is t = 36/V = 36/(v + v').

he travels 32 km downstream, the time taken is t' = 32/V' = 32/(v - v')

The total time is thus t + t' = 36/(v + v') + 32/(v - v')

Since the whole trip takes 6,5 hours,

36/(v + v') + 32/(v - v') = 6.5  (1)

Multiplying each term by (v + v')(v - v'), we have

(v + v')(v - v')36/(v + v') + (v + v')(v - v')32/(v - v') = 6.5(v + v')(v - v')  (1)

(v - v')36 + (v + v')32 = 6.5(v + v')(v - v')  (1)

36v - 36v' + 32v + 32v' = 6.5(v² + v'²)

68v - 4v' = 6.5(v² + v'²)        (2)

Also he travels 4 km upstream, the time taken is t" = 4/V = 4/(v + v').

he travels 40 km downstream, the time taken is t'" = 40/V' = 40/(v - v')

The total time is thus t" + t'" = 4/(v + v') + 40/(v - v')

Since the whole trip takes 180 minutes = 3 hours,

4/(v + v') + 40/(v - v') = 3  (3)

Multiplying each term by (v + v')(v - v'), we have

(v + v')(v - v')4/(v + v') + (v + v')(v - v')40/(v - v') = 3(v + v')(v - v')  (1)

(v - v')4 + (v + v')40 = 3(v + v')(v - v')  (1)

4v - 4v' + 40v + 40v' = 3(v² + v'²)

44v - 36v' = 3(v² + v'²)      (4)

Dividing (2) by (4), we have

(68v - 4v')/(44v - 36v') = 6.5(v² + v'²)/3(v² + v'²)      

(68v - 4v')/(44v - 36v') = 6.5/3

3(68v - 4v') = 6.5(44v - 36v')

204v - 12v' = 286v - 234v'

204v - 286v = 12v' - 234v'

-82v = -222v'

v = -222v'/82

v = 111v'/41

Substituting v into (2), we have

68v - 4v' = 6.5(v² + v'²)      

68(111v'/41) - 4v' = 6.5[(111v'/41)² + v'²]        

[68(111/41) - 4]v' = 6.5[(111/41)² + 1]v'²    

[68(111/41) - 4]v' = 6.5[(111/41)² + 1]v'²

[7548/41 - 4]v' = 6.5[12321/1681 + 1]v'²

[(7548 - 164)/41]v' = 6.5[(12321 + 1681)/1681]v'²

[7384/41]v' = 6.5[14002/1681]v'²

[7384/41]v' = [91013/1681]v'²

[91013/1681]v'² - [7384/41]v' = 0

([91013/1681]v' - [7384/41])v' = 0

⇒ v' = 0 or ([91013/1681]v' - 7384/41) = 0

⇒ v' = 0 or [91013/1681]v' - 7384/41) = 0

⇒ v' = 0 or v' =  7384/41 × 16841/91013

⇒ v' = 0 or v' =  180.097 × 0.185

⇒ v' = 0 or v' =  33.33 km/h

Since v' ≠ 0, v' = 33.33 km/h

Substituting v' into v = 111v'/41 = 111(33.33 km/h)/41 = 3699.63 km/h ÷ 41 = 90.23 km/h

So, the streamer's speed in still water is 90.23 km/h while the stream's speed is 33.33 km/h

Brad bought a piece of industrial real estate for $192,345. The value of the real estate appreciated a constant rate per year. The table shows the value of the real estate after the first and second years:

Year:
1
2

Value (in dollars):
$200,038.80
$208,040.35

Which function best represents the value of the real estate after t years?
A. f(t) = 200,038.80(1.04)^t

B. f(t) = 200,038.80(0.04)^t

C. f(t) = 192,345(0.04)^t

D. f(t) = 192,345(1.04)^t

Answers

Answer:

D. f(t) = 192,345(1.04)^t

Step-by-step explanation:

I took the test and it was right.

Also that is the original price and when you look at exponential functions, the starting point or original price is always first the then rate of increase. The table just shows how it increased in year 1 and 2.

Hope this helps. :)

A function assigns the values. The function that best represents the value of the real estate after t years is f(t) = 192,345(1.04)^t. Thus, the correct option is D.

What is a Function?

A function assigns the value of each element of one set to the other specific element of another set.

The initial cost of industrial real estate is $192,345, while the cost after one year is $200,038.80. Therefore, the rate of appreciation is,

[tex]\$200,038.80 = \$192,345(1+R)^t\\\\\$200,038.80 = \$192,345(1+R)^1\\\\\dfrac{\$200,038.80}{\$192,345}=(1+R)^t\\\\1.04 = 1 + R\\\\R = 0.04[/tex]

Hence, the function that best represents the value of the real estate after t years is f(t) = 192,345(1.04)^t. Thus, the correct option is D.

Learn more about Function:

https://brainly.com/question/5245372

#SPJ2

provide explanation too please !


a piano teacher teaches 8 lessons in 6 hours.
the lessons are all the same length.
how long is one lesson, in minutes?

Answers

1 hour is 60 minutes

6 hours x 60 = 360 total minutes

Divide total time by number of lessons:

360/8 = 45

Each lesson was 45 minutes

Complete the equation for the relationship between the weight and number of months

Answers

Y=5.5x because it just is

Find the slope of the line that passes through (2 2) and (-1 -2)

Answers

Answer:

4/3

Step-by-step explanation:

The formula is (y1-y2) /(x1-x2)

Answer:

slope = [tex]\frac{4}{3}[/tex]

Step-by-step explanation:

Calculate the slope m using the slope formula

m = [tex]\frac{y_{2}-y_{1} }{x_{2}-x_{1} }[/tex]

with (x₁, y₁ ) = (2, 2) and (x₂, y₂ ) = (- 1, - 2)

m = [tex]\frac{-2-2}{-1-2}[/tex] = [tex]\frac{-4}{-3}[/tex] = [tex]\frac{4}{3}[/tex]

a box contains 6 dimes, 8 nickels, 12 pennies, and 3 quarters. what is the probability that a coin drawn at random is not a dime

Answers

Add all the coins to get total coins:

6 + 8 + 12 + 3 = 29 total coins

Subtract dimes to find total of the coins that are not dimes:

29 -6 = 23

Probability of not picking a dime is the number of coins that aren’t dimes over total coins:

23/29

Which numbers are divisible by 2?

A)826

B)270

C)271

Answers

Answer:

A

Step-by-step explanation:

Answer:

A and B, every even number is divisible by two, C is odd

Put these numbers in order from least to greatest.
9.9, 9.5, and 9 4/5

Answers

9.5, 9 4/5, 9.9

9 4/5 is equal to 9.8

Given the interval 0<θ<π/2. Find the angle θ which is formed by the line y = -2x+4 and y = 3x-3
Show your work as well, thank you!​

Answers

Answer:

[tex]\rm\displaystyle \theta = \frac{\pi}{4} [/tex]

Step-by-step explanation:

we want to find the acute angle θ (as θ is between (0,π/2)) formed by the line y=-2x+4 and y=3x-3 to do so we can consider the following formula:

[tex] \displaystyle\tan( \theta) = \bigg| \frac{ m_{2} - m_{1} }{1 + m_{1} m_{2} } \bigg | [/tex]

[tex] \rm \displaystyle \implies\theta = \arctan \left( \bigg | \frac{ m_{2} - m_{1} }{1 + m_{1} m_{2} } \bigg | \right)[/tex]

From the first equation we obtain that [tex]m_1[/tex] is -2 and from the second that [tex]m_2[/tex] is 3 therefore substitute:

[tex] \rm\displaystyle \theta = \arctan \left( \bigg | \frac{ 3 - ( - 2) }{1 + ( - 2) (3)} \bigg | \right)[/tex]

simplify multiplication:

[tex] \rm\displaystyle \theta = \arctan \left( \bigg | \frac{ 3 - ( - 2) }{1 + ( - 6)} \bigg | \right)[/tex]

simplify Parentheses:

[tex] \rm\displaystyle \theta = \arctan \left( \bigg | \frac{ 3 + 2 }{1 + ( - 6)} \bigg | \right)[/tex]

simplify addition:

[tex] \rm\displaystyle \theta = \arctan \left( \bigg | \frac{ 5 }{ - 5} \bigg | \right)[/tex]

simplify division:

[tex] \rm\displaystyle \theta = \arctan \left( | - 1| \right)[/tex]

calculate the absolute of -1:

[tex]\rm\displaystyle \theta = \arctan \left( 1\right)[/tex]

calculate the inverse function:

[tex]\rm\displaystyle \theta = \frac{\pi}{4} [/tex]

hence,

the angle θ which is formed by the line y = -2x+4 and y = 3x-3 is π/4

(for more info about the formula refer the attachment thank you!)

Consider two vector-valued functions,

[tex]\vec r(t) = \left\langle t, -2t+4\right\rangle \text{ and } \vec s(t) = \left\langle t, 3t-3\right\rangle[/tex]

Differentiate both to get the corresponding tangent/direction vectors:

[tex]\dfrac{\mathrm d\vec r(t)}{\mathrm dt} = \left\langle1,-2\right\rangle \text{ and } \dfrac{\mathrm d\vec s(t)}{\mathrm dt} = \left\langle1,3\right\rangle[/tex]

Recall the dot product identity: for two vectors [tex]\vec a[/tex] and [tex]\vec b[/tex], we have

[tex]\vec a \cdot \vec b = \|\vec a\| \|\vec b\| \cos(\theta)[/tex]

where [tex]\theta[/tex] is the angle between them.

We have

[tex]\langle1,-2\rangle \cdot \langle1,3\rangle = \|\langle1,-2\rangle\| \|\langle1,3\rangle\| \cos(\theta) \\\\ 1\times1 + (-2)\times3 = \sqrt{1^2 + (-2)^2} \times \sqrt{1^2+3^2} \cos(\theta) \\\\ \cos(\theta) = \dfrac{-5}{\sqrt5\times\sqrt{10}} = -\dfrac1{\sqrt2} \\\\ \implies \theta = \cos^{-1}\left(-\dfrac1{\sqrt2}\right) = \dfrac{3\pi}4[/tex]

Then the acute angle between the lines is π/4.

Kayleigh has $4500 in a savings account at the bank that earns 0.8% interest per year. How much
interest will she earn in 3 years?

Answers

Answer:

Kayleigh will have a total of $4608.

Step-by-step explanation:

First, you use the formula, I=PRT (Interest=Principal, Rate, Time), then you distribute the numbers: (I=4500x0.8%x3) when you multiply them all, you get $108, then you lastly add 108 to 4500, and you get your final answer of $4608.

Kayleigh will earn $108 in interest over a period of 3 years.

Interest is the additional amount of money that is charged or earned on a principal amount of money. It is typically expressed as a percentage of the principal and is either charged when borrowing money or earned when investing or saving money.

To calculate the interest Kayleigh will earn in 3 years, we can use the formula for simple interest:

Interest = Principal x  Rate x Time

Given:

Principal = $4500

Rate = 0.8% = 0.008 (decimal form)

Time = 3 years

Plugging the values into the formula, we have:

Interest = $4500 x 0.008 x 3

Calculating the expression, we find:

Interest = $108

Therefore, Kayleigh will earn $108 in interest over a period of 3 years.

To know more about simple interests follow

https://brainly.com/question/29785550

#SPJ2

wich is a solution to (x-3)(x+9)=-27?​

Answers

Answer:

=0=−6

Step-by-step explanation:

If R(x) = 2 – 3x – 1, find R(-2)

a. -3
b. 9
c. -9
d. -11

PLEASE HELP

Answers

Answer:

C

Step-by-step explanation:

C

The output of  [tex]R(x) = x^{2} - 3x- 1[/tex] when x = -2 is 9.

What is PEMDAS?

PEMDAS exists as an acronym for the terms parenthesis, exponents, multiplication, division, addition, and subtraction.

To estimate the value of R(-2)

Substitute the value of x = -2, then

[tex]R(-2) = (-2)^{2} - 3(-2) - 1[/tex]

By using the PEMDAS order of operations

Calculate exponents, [tex](-2)^2 = 4[/tex]

= 4 - 3(-2) - 1

Multiply and divide (left to right), 3(-2) = -6

= 4 - (-6) - 1

Add and subtract (left to right),

4 - (-6) - 1 = 9

Therefore, the value of R(-2) = 9.

To learn more about the value of a function refer to:

https://brainly.com/question/15712340

#SPJ2

Other Questions
Match the healthcare facility to the client.1. hospice2. rehabilitation center3. public hospital4. mental health facilityA. Nita suffers from severe depression.B. Mike has been diagnosed with stage 4 cancer,and the doctor has told him that he has just afew months to live.C. Janice has an alcohol addiction.D. Yasmin has injured her leg in a car accident. Directions: Read the following statements and write your counterclaim.1. Students' grades really measure achievement.2. Smoking causes lung cancer.3. One parent argues that television has a negative effect on family life.4. Heavy snoring is dangerous.5. A lie detector test is not really accurate. 7a - 2b = 5a + ba = 2ba = 3ba = a equals StartFraction 3 Over 2 EndFraction b.ba = a equals StartFraction 2 Over 3 EndFraction b.b Help yall Cross Sections of a Cone: Introduce possible cross sections of a cone. You need to show the shape of the cross section and how you cut the cone to get that shape. Please introduce at least THREE AJ's Markets is being liquidated. The mortgage holder is owed $830,000, the other secured creditors are owed $128,000, and the unsecured creditors are owed $329,000. Administrative costs of liquidation, wage and benefit payments, and consumer claims amount to $330,000. The firm owes no taxes. The building, which is mortgaged, just netted $794,000 after sale costs. The remaining assets have yielded $467,000 in net proceeds. How much will the unsecured creditors receive per each dollar they are owed?A. $.027B. $.025C. $.333D. $1.00E. $.533 Complete the sentences with the prepositions or de.1. Vous pouvez ----- travailler le week-end.2. Vous savez ----- parler plusieurs langues. Last week, the price of apples at a grocery store was $1.60 per pound. This week, apples at the same grocery store are on sale at a 10% discount. What is the total price of 4 \frac{ 1}{2 } 21 pounds of apples this week at the grocery store? Calculate the heat energy, in kJ, needed to increase the temperature of 375g of water by 45.0C. Claim 2: The fertilizer changed into rust. Is this claim supported or refuted? Why? Based on these excerpts, how do the Maori and Haidaview the spirits of the sky? When was this created? HELP ASAP< BRAINLIEST IF RIGHT Subtract to find the temperature changesfor the water and the metal. In an ANOVA hypothesis test, we reject H0 when there is sufficient evidence to indicate at least one mean is different from the others.a. Trueb. False 2 Pointsof 20Which word refers to the medium of a story?O A. PersonificationB. BalladC. Comic bookD. SymbolSUBMI In a survey of 1000 people, 700 people said that they voted in the last presidential election. Let p denote the proportion of all people who voted. Find a point estimate for p and also construct a 90% confidence interval for p. a) 0.700, (0.676, 0.724) b) 700, (0.676,0.724) c) 700, (0.642,0.767) d) 0.700, (0.642,0.767) e) 0.300, (0.276, 0.324) If a chair costs 734 dollars and I give it to my friend for 823 dollars how much is my profit as a percentage the perimeter of a circular segment with a central angle of 144 degrees is 30 cm calculate the area of the segment to 2 decimal places Please solve the following equation. WILL MARK BRAINLIEST AND NO RUBBISH LINKS OR ANSWERS. Thank you!38x-12y = (6+2n)x + (4-n)y Explain the significance and impact of the American with Disabilities Act on businesses in the United States.